least common multiple

3/6 , 2/12

Answers

Answer 1

Answer:

6

Step-by-step explanation:

3,6,9.12

2,4,6,8,10,12


Related Questions

Select the correct answer from each drop-down menu. Gino is buying wood screws at the corner hardware store. The table shows different numbers of bags of screws and their corresponding prices. Bags of Screws Price ($) 2 10 4 20 7 35 According to the table, the relationship between the number of bags and the price is proportional or not proportional

Answers

Without the table I will guess 5 is common to all 3 they are proportional

Divide 36x5 − 44x4 − 28x3 by 4x2.

Answers

Answer:-

36*5= 180

44*4= 176

28*3= 84

4*2=8

180-176-84= (-80)

-80/8=(-10)

Step-by-step explanation: Hope this helps. Mark me brainliest :)))

2) √51 is closest to which whole
number?

Answers

After cοmpleting the task, we can state that The whοle number clοsest tο  expressiοn 7.141 is 7. Therefοre, √51 is clοsest tο the whοle number 7.

What is whοle number?

The whοle numbers are the part οf the number system which includes all the pοsitive integers frοm 0 tο infinity. These numbers exist in the number line. Hence, they are all real numbers. We can say, all the whοle numbers are real numbers, but nοt all the real numbers are whοle numbers.

Thus, we can define whοle numbers as the set οf natural numbers and 0. Integers are the set οf whοle numbers and negative οf natural numbers. Hence, integers include bοth pοsitive and negative numbers including 0. Real numbers are the set οf all these types οf numbers, i.e., natural numbers, whοle numbers, integers and fractiοns.

√51 is apprοximately equal tο 7.141. The whοle number clοsest tο 7.141 is 7.

Therefοre, √51 is clοsest tο the whοle number 7.

To know more about whole number visit :-

https://brainly.com/question/29766862

#SPJ1

a scientists who studies insects enlarges a photograph of an elm leaf beetle every 2 inches in the photograph represents 8mm on the actual beetle the length of the beetle in the photograph is 1 1/2 inches .the width of the beetle in the photograph 3/4 inches what are the length and the
width of the actual beetle show your work

Answers

The length of the beetle is 6mm and the width is 3mm

A enlarged drawing is a drawing that is bigger than the original image / building / object

the enlarged drawing is usually increased at a constant dimension

Scale of the enlarged drawing = enlarged dimensions / original dimension

2 inches : 8 mm

1 : 4

So, 1 inch represents 4 millimetres

Length of the actual beetle = \(1\frac{1}{2}\) x 4

= \(\frac{3}{2}\) x 4 = 6mm

Width of the actual beetle = \(\frac{3}{4}\) x 4 = 3mm

A similar question was solved here: https://brainly.com/question/18291941?referrer=searchResults

An isosceles triangle has an angle that measures 136°. What measures are possible for the other two angles? Choose all that apply.

Answers

Answer:

The other two angles are 22° , 22°

Step-by-step explanation:

To find the other angles, we can use angle sum property of triangle.

      The given angle 136° cannot be base angles. Let the base angles be x.

x + x + 136 = 180°

   2x + 136 = 180°

Subtract 136 from both sides,

            2x = 180 - 136

            2x = 44°

Divide both sides by 2,

              x = 44 ÷ 2

                \(\sf \boxed{x = 22^\circ}\)

Answer:42

Step-by-step explanation:

just add

Kelsey has a list of possible functions. Pick one of the g(x) functions below and then describe to Kelsey the key features of g(x), including the end behavior, y-intercept, and zeros.
g(x) = (x + 2)(x − 1)(x − 2)
g(x) = (x + 3)(x + 2)(x − 3)
g(x) = (x + 2)(x − 2)(x − 3)
g(x) = (x + 5)(x + 2)(x − 5)
g(x) = (x + 7)(x + 1)(x − 1)

Answers

The zeros of g(x) = (x + 2)(x − 1)(x − 2) are x = -2, 1 and 2

The y-intercept is g(0) = 4The end behaviour is \(\mathrm{as}\:x\to \:+\infty \:,\:g\left(x\right)\to \:+\infty \:,\:\:\mathrm{and\:as}\:x\to \:-\infty \:,\:g\left(x\right)\to \:-\infty \:\)
Calculating the key features of the function g(x)

From the question, we have the following parameters that can be used in our computation:

The 5 functions of g(x)

To determine the key features of the function g(x), we make use of the first

g(x) = (x + 2)(x − 1)(x − 2)

So, we have

Zeros

This is when the function equals 0

(x + 2)(x − 1)(x − 2) = 0

Evaluate

x = -2, 1 and 2

The y-intercept

This is when the value of x in the function equals 0

g(0) = (0 + 2)(0 − 1)(0 − 2)

Evaluate

g(0) = 4

End behaviour

This is the behavior of the graph of the function as it approaches the ends of the x-axis

So, we have

\(\mathrm{as}\:x\to \:+\infty \:,\:g\left(x\right)\to \:+\infty \:,\:\:\mathrm{and\:as}\:x\to \:-\infty \:,\:g\left(x\right)\to \:-\infty \:\)

Read more about polynomial at

https://brainly.com/question/30833611

#SPJ1

Mathematics

12g - 16 =

Answers

Answer:

g= -4 is correct answer

You would like to have $20,000 to use a down payment for a home in five years by making regular, end-of-month deposits into an annuity that pays 6% interest compounded monthly.


How much should you deposit each month?




Round your answer to the nearest cent. Do not include the dollar sign in the answer box below.

You would like to have $20,000 to use a down payment for a home in five years by making regular, end-of-month

Answers

The calculation of this can be done by first determining the future value of the monthly payments of $327.50

The future value of an annuity can be determined using a financial calculator, mathematical formula, or spreadsheet software. The future value of an annuity is calculated by multiplying the periodic payment amount by the future value factor,

which is based on the number of payments and the interest rate.For example, suppose we want to know the future value of a $500 end-of-month deposit into an annuity that pays 6% interest compounded monthly for five years.

The future value factor for 60 periods at 0.5 percent per month is 80.9747, which can be multiplied by the monthly deposit amount to find the future value of the annuity.500 × 80.9747 = 40,487.35

This means that a $500 end-of-month deposit into an annuity paying 6% interest compounded monthly for five years will have a future value of $40,487.35.

Therefore, to accumulate a $20,000 down payment for a home in five years, you would need to deposit $327.50 per month into the annuity.

 for 60 months using the formula and then solving for the monthly payment amount where FV = $20,000 and n = 60, r = 0.5%.FV = PMT [(1 + r)n – 1] / r$20,000 = PMT [(1 + 0.005)60 – 1] / 0.005PMT = $327.50 (rounded to the nearest cent).

To learn more about : calculation

https://brainly.com/question/17145398

#SPJ8

5 red beads,3 blue,4 green probability of a blue bead

Answers

Answer:

3/12 or 40%

not 10000% sure on this tho

the marked price of an article is 35% more than that of cost price.If the article is sold allowing 15% discount,profit becimes Rs.200.Find the MP and Cp of the article.​

Answers

Answer:

B

Step-by-step explanation:

Write the answer and solution ?

Write the answer and solution ?

Answers

Answer

4. E

use PEMDAS

5. 36+18=54  so C

plug in x

6. 15=5x

x=3 so C

11. B

break the absolute value into pos and neg components

12.D

is not equal to so open circle

2x=14

2x=8

so 4 and 7 with open circles

Step-by-step explanation:

What is m% of n? Can you please write an explanation on how that is the answer?

Answers

Answer:

mn/100

Step-by-step explanation:

m% of n

\( = \frac{m}{100} \times n \\ \\ = \frac{mn}{100} \)

Donna is participating in a 6-day cross-country biking challenge. She biked for 51, 62, 61, 47, and 63 miles on the first five days. How many miles does she need to bike on the last day so that her average (mean) is 56 miles per day?

Answers

Answer:

52 miles

Step-by-step explanation:

total miles needed to average 56

56 * 6 = 336

miles after 5 days

51 + 62 + 61 + 47 + 63 = 284

miles needed on last day

336 - 284 = 52 miles

825 use each digit once. make the smallest 3digit number​

Answers

Step-by-step explanation:

Given: To make smallest 3-digit number of 825.

To find: The smallest 3-digit number of 825.

Solution: We can make the smallest 3-digit number of 825 by separating the numbers and arranging it to ascending order. The given number is 825. ...

Final answer: The smallest 3-digit number of 825 is 258.

hope it helps

Answer:

258

Step-by-step explanation:

We are given 3 numbers:

8 2 5

And we are asked to find the smallest 3 digit number using those 3 digits above.

To make the smallest number, place the numbers in value from least to greatest:

2 5 8

This is your 3 digit number: 258.

Hope this helps! :)

1)
Science 5
Measurement Assignment
Name each measurement instrument below. Then, indicate which
type of measurement is performed with each one. Remember,
some instruments can be used for more than one type of
measurement!
Instrument
Name of Instrument
Measurement Type

1)Science 5Measurement AssignmentName each measurement instrument below. Then, indicate whichtype of

Answers

Answer: See below

Step-by-step explanation:

The first is a beaker, it's used to measure liquid volume

The second is a ruler, it's used to measure length.

Last is a thermometer, it's used to measure temperature.

Write an expression to represent the
total area as the sum of the areas of
each room.
11(7 + 4) =
? · 7+11.
?

Write an expression to represent thetotal area as the sum of the areas ofeach room.11(7 + 4) =? 7+11.?

Answers

11 x 7 + 11 x 4 is the expression of the total area as the sum of the areas of

each room.

What is an expression?

An expression is a way of writing a statement with more than two variables or numbers with operations such as addition, subtraction, multiplication, and division.

Example: 2 + 3x + 4y = 7 is an expression.

We have,

11 (7 + 4)

Using the property of multiplication over addition.

11 x 7 + 11 x 4

Thus,

11 x 7 + 11 x 4.

Learn more about expressions here:

https://brainly.com/question/3118662

#SPJ9

Lina placed $1000 in a bank account with an annual compound interest rate of 8%.
How long will it take until she has $5000 in the account?

Answers

After 50 years, she gets the amount $5000 in her bank account.

According to the statement

we have given that the Lina placed $1000 in a bank account with an annual compound interest rate of 8%.

And we have to find that the How long will it take until she has $5000 in the account.

So, For this purpose, we know that the

The interest rate is the amount a lender charges a borrower and is a percentage of the principal—the amount loaned.

The principal amount = 1000

And 8% of P.A. = 8/100*1000

8% of P.A. = 80.

So, According to this after 1 year the amount become

1080.

it means the equation become after t years.

Amount(t) = 1000 + 80t

And the amount is 5000 which is given then

5000 = 1000 + 80t

4000 = 80t

t = 50.

So, After 50 years, she gets the amount $5000 in her bank account.

Learn more about interest rate here

https://brainly.com/question/25545513

#SPJ1

Find the quotient of z₁ by z2. Express your answer in
trigonometric
form.
² - 3 (0 (4) + (*))
Z₁ cos
+/sin
Z₂
²2 = 7 (cos(377)+
COS
8
O A. 7 (cos (577) + i sin (5/77))
8
B.
21(cos(577)+isin (577))
8
OC. 21 cos
21(cos(-7)+ i sin(-77))
O D. 7 (cos(-7) + + sin(-7))
i
+/sin
37T
8

Find the quotient of z by z2. Express your answer intrigonometricform. - 3 (0 (4) + (*))Z cos+/sinZ2

Answers

The quotient of z₁ by z₂ in trigonometric form is:

7/21 * (cos(584°) + i sin(584°))

To find the quotient of z₁ by z₂ in trigonometric form, we'll express both complex numbers in trigonometric form and then divide them.

Let's represent z₁ in trigonometric form as z₁ = r₁(cosθ₁ + isinθ₁), where r₁ is the magnitude of z₁ and θ₁ is the argument of z₁.

We have:

z₁ = 7(cos(577°) + i sin(577°))

Now, let's represent z₂ in trigonometric form as z₂ = r₂(cosθ₂ + isinθ₂), where r₂ is the magnitude of z₂ and θ₂ is the argument of z₂.

From the given information, we have:

z₂ = 21(cos(-7°) + i sin(-77°))

To find the quotient, we divide z₁ by z₂:

z₁ / z₂ = (r₁/r₂) * [cos(θ₁ - θ₂) + i sin(θ₁ - θ₂)]

Substituting the given values, we have:

z₁ / z₂ = (7/21) * [cos(577° - (-7°)) + i sin(577° - (-7°))]

= (7/21) * [cos(584°) + i sin(584°)]

The quotient of z₁ by z₂ in trigonometric form is:

7/21 * (cos(584°) + i sin(584°))

Option C, 21(cos(-7°) + i sin(-77°)), is not the correct answer as it does not represent the quotient of z₁ by z₂.

For more questions on trigonometric form

https://brainly.com/question/31744474

#SPJ8

Enter the number
10.333333333333
rounded to the nearest hundredth (two decimal places):

Answers

Answer:

Step-by-step explanation:

This decimal rounded to the nearest 1/100 is 10.33 The next 3 in the 1/1000 place, is not large enough to influence the 3 in the hundredth place.

So the answer is 10.33

Which linear function has the same y-intercept as the one that is represented by the graph?

On a coordinate plane, a line goes through points (3, 4) and (5, 0).
A 2-column table with 4 rows. Column 1 is labeled x with entries negative 3, negative 1, 1, 3. Column 2 is labeled y with entries negative 4, 2, 8, 14.
A 2-column table with 4 rows. Column 1 is labeled x with entries negative 4, negative 2, 2, 4. Column 2 is labeled y with entries negative 26, negative 18, negative 2, 6.
A 2-column table with 4 rows. Column 1 is labeled x with entries negative 5, negative 3, 3, 5. Column 2 is labeled y with entries negative 15, negative 11, 1, 5.
A 2-column table with 4 rows. Column 1 is labeled x with entries negative 6, negative 4, 4, 6. Column 2 is lab
eled y with entries negative 26, negative 14, 34, 46.

Answers

The linear function that has the same y-intercept as the given graph is the equation y = -2x + 10, corresponding to option 3.

To determine the linear function with the same y-intercept as the graph, we need to find the equation of the line passing through the points (3, 4) and (5, 0).

First, let's find the slope of the line using the formula:

slope (m) = (change in y) / (change in x)

m = (0 - 4) / (5 - 3)

m = -4 / 2

m = -2

Now that we have the slope, we can use the point-slope form of a linear equation to find the equation of the line:

y - y1 = m(x - x1)

Using the point (3, 4) as our reference point, we have:

y - 4 = -2(x - 3)

Expanding the equation:

y - 4 = -2x + 6

Simplifying:

y = -2x + 10

Now, let's check the given options to find the linear function with the same y-intercept:

Option 1: The table with x-values (-3, -1, 1, 3) and y-values (-4, 2, 8, 14)

The y-intercept is not the same as the given line. So, this option is not correct.

Option 2: The table with x-values (-4, -2, 2, 4) and y-values (-26, -18, -2, 6)

The y-intercept is not the same as the given line. So, this option is not correct.

Option 3: The table with x-values (-5, -3, 3, 5) and y-values (-15, -11, 1, 5)

The y-intercept is the same as the given line (10). So, this option is correct.

Option 4: The table with x-values (-6, -4, 4, 6) and y-values (-26, -14, 34, 46)

The y-intercept is not the same as the given line. So, this option is not correct.

Therefore, the linear function that has the same y-intercept as the given graph is the equation y = -2x + 10, corresponding to option 3.

for such more question on linear function

https://brainly.com/question/9753782

#SPJ8

A triangular window has an area of 594 square meters. The base is 54 meters. What is the height?

Answers

Answer:

  22 m

Step-by-step explanation:

Use the formula for the area of a triangle. Fill in the known values and solve for the unknown.

  A = (1/2)bh

  594 m^2 = (1/2)(54 m)h

  h = (594 m^2)/(27 m) = 22 m

The height of the window is 22 meters.

How do I do this I am a bit confused ?

How do I do this I am a bit confused ?

Answers

When you multiply the same number with different exponents together, you add the exponents together.

3^2 + 3^x = 3^7

Adding the two together you have 3^(2 + x = 7)

Solve for x: 2 + x = 7

Subtract 2 from both sides:

x = 5

Find the perimeter of this L shape.

Find the perimeter of this L shape.

Answers

Answer: undefined, not enough information

Step-by-step explanation:

Answer:

15+19 =34 is the perimeter of the closed figure

hope it helped you

5 apples cost 60p how much does 1 apple cost

Answers

Answer:

It will cost 12p a one apple

You have relatives living in the United Kingdom and in France. Suppose that you have purchased a prepaid phone card with a value of $75. Calls to the United Kingdom cost $0.23 per minute, while calls to France cost $0.21 per minute.
(a) Write a linear equation in two variables to represent the number of minutes you can use to call those two locations.
(b) Graph the inequality, and discuss a possible solution in the context of the real-life situation.

Answers

Answer:

a) 0.23x+0.21y=75

b) (For the Graph see the attached picture).

A possible solution for the inequality \(0.23x+0.21y\leq75\) would be any point inside the shaded region of the graph. For example (150,175) This is 150 minutes to the United Kingdom and 175 minutes to France.

\(0.23x+0.21y\leq75\)

\(0.23(150)+0.21(175)\leq75\)

\(34.5+36.75\leq75\)

\(71.25\leq 75\)

this inequality is true, so the number of minutes used for the United Kingdom and to France is valid.

Step-by-step explanation:

a)

In order to solve this problem, we must first set our variables:

x= Minutes to the United Kingdom.

y= Minutes to France

The greatest amount of money you can spend is $75 and each minute will cost $0.23 when calling to the United Kingdom and $0.21 when calling to France. So we can use this information to build our equation:

0.23x+0.21y=75.

b) So first, we need to convert our equation into an inequality where the total amount of money spent must be less than $75, so our inequality is:

[tex}0.23x+0.21y\leq75\)

so now we can proceed and graph. This is graphed exactly as you would graph a regular linear equation. You need to find two points on the graph that will satisfy the equation. Plot them and then connect them with a straight line. For example:

First, let's solve the equation for y:

0.23x+0.21y=75

we start by moving the 0.23x to the other side of the equation so we get:

0.21y=-0.23x+75

and next we divide both sides of the equation into 0.21 so we get:

\(y=\frac{-0.23x+75}{0.21}\)

which yields:

y= -1.095x+357.14

next we need to pic an x-value so we can find the first ordered pair. Let's say I pick x=0. So we get:

y= -1.095x+357.14

y= -1.095(0)+357.14

y=357.14

so our first point is (0, 357.14)

And we can follow the same procedure for the second point. Let's say I pick x=1. In that case our second point is (1, 354.04). We can now plot them. Once the graph is drawn, we need to shade it, for which we will pick an ordered pair to the left and an ordered pair to the right of the line. For the left region let's pick the point (0,0) and for the right of the graph, let's pick the point (150,357).

So let's test the inequality for these two points:

First, let's use the point (0,0)

\(0.23x+0.21y\leq75\)

\(0.23(0)+0.21(0)\leq75\)

\(0\leq75\)

This proves that the left side of the graph is the side to be shaded. We can still use the other point and see what we qet:

(150, 357) and let's use it on our inequality:

\(0.23x+0.21y\leq75\)

\(0.23(150)+0.21(357)\leq75\)

\(109.47\leq75\)

Is a false statement, so only the region on the left will contain the possible number of minutes to do the phone calls to the UK and France.

A possible solution for the inequality \(0.23x+0.21y\leq75\) would be any point inside the shaded region of the graph. For example (150,175) This is 150 minutes to the United Kingdom and 175 minutes to France.

\(0.23x+0.21y\leq75\)

\(0.23(150)+0.21(175)\leq75\)

\(34.5+36.75\leq75\)

\(71.25\leq 75\)

This is a true statement so the possible solution is correct.

You have relatives living in the United Kingdom and in France. Suppose that you have purchased a prepaid

Enter the number that belongs in the green box

Enter the number that belongs in the green box

Answers

The angle between the sides measuring 4 and 5 in the obtuse triangle is approximately 101.54 degrees.

To find the measure of the angle between the sides measuring 4 and 5 in an obtuse triangle with side lengths 4, 5, and 7, we can use the Law of Cosines. The Law of Cosines states that in a triangle with side lengths a, b, and c, and an angle opposite to side c, the following equation holds:

\(c^2 = a^2 + b^2 - 2ab*cos(C)\)

In this case, we have side lengths a = 4, b = 5, and c = 7. We want to find the angle C, which is opposite to side c. Substituting these values into the Law of Cosines, we get:

\(7^2 = 4^2 + 5^2\)- 2(4)(5)*cos(C)

49 = 16 + 25 - 40*cos(C)

49 = 41 - 40*cos(C)

40*cos(C) = 41 - 49

40*cos(C) = -8

cos(C) = -8/40

cos(C) = -0.2

To find the measure of angle C, we can take the inverse cosine (arccos) of -0.2:

C = arccos(-0.2)

Using a calculator, we find that C ≈ 101.54 degrees.

Therefore, the measure of the angle between the sides measuring 4 and 5 in the obtuse triangle is approximately 101.54 degrees.

For more such information on: angle

https://brainly.com/question/25716982

#SPJ8

What is the answer to this question?

What is the answer to this question?

Answers

Rewrite 1/2 as 4/8

Now add 5 3/8 + 2 4/8

Answer: 7 7/8

I was confused on these
T-T

I was confused on these T-T

Answers

Answer:

Step-by-step explanation: The answer for 7: is 7/2

8: 28/5.

9: 9/4

10: 24/5

I need explanation for example 8.
Thankyou

I need explanation for example 8.Thankyou

Answers

There is a probability of 94/315 that the problem will be solved.

We are given that P has a chance of solving the problem of 2/7, Q has a chance of solving the problem of 4/7, and R has a chance of solving the problem of 4/9. To find the probability that the problem is solved, we need to consider all possible scenarios in which the problem can be solved.

The probability of this scenario is 2/7. If P solves the problem, then it does not matter whether Q or R solve it, the problem is already solved. Therefore, the probability of the problem being solved in this scenario is 2/7.

The probability of this scenario is 4/7. If Q solves the problem, then it does not matter whether P or R solve it, the problem is already solved. Therefore, the probability of the problem being solved in this scenario is 4/7.

The probability of this scenario is 4/9. If R solves the problem, then it does not matter whether P or Q solve it, the problem is already solved. Therefore, the probability of the problem being solved in this scenario is 4/9.

The probability of this scenario is (1-2/7) * (1-4/7) * (1-4/9) = 3/35. This is because the probability of P not solving the problem is 1-2/7, the probability of Q not solving the problem is 1-4/7, and the probability of R not solving the problem is 1-4/9. To find the probability of none of them solving the problem, we multiply these probabilities together.

To find the probability of the problem being solved, we need to add the probabilities of all the scenarios in which the problem is solved. Therefore, the probability of the problem being solved is:

2/7 + 4/7 + 4/9 = 94/315

To know more about probability here

https://brainly.com/question/11234923

#SPJ1

Given the function f(x) = 0.5|x - 41-3, for what values of x is f(x) = 7?
x = -24, x = 16
x= -16, x = 24
x=-1, x = 9
x = 1, x = -9

Answers

The values of x for which f(x) = 7 are x = 61 and x = 21.

To find the values of x for which f(x) = 7, we can set up the equation and solve for x.

The given function is f(x) = 0.5|x - 41| - 3.

Setting f(x) equal to 7, we have:

0.5|x - 41| - 3 = 7.

First, let's isolate the absolute value term:

0.5|x - 41| = 7 + 3.

0.5|x - 41| = 10.

To remove the absolute value, we can consider two cases:

Case: (x - 41) is positive or zero:

0.5(x - 41) = 10.

Multiplying both sides by 2 to get rid of the fraction:

x - 41 = 20.

Adding 41 to both sides:

x = 61.

So x = 61 is a solution for this case.

Case: (x - 41) is negative:

0.5(-x + 41) = 10.

Multiplying both sides by 2:

-x + 41 = 20.

Subtracting 41 from both sides:

-x = -21.

Multiplying both sides by -1 to solve for x:

x = 21.

So x = 21 is a solution for this case.

Therefore, the values of x for which f(x) = 7 are x = 61 and x = 21.

for such more question on values

https://brainly.com/question/11546044

#SPJ8

Other Questions
Can anyone help in this question...... Describe the following pattern in your own word and write down three next terms:1;3;5;7;9 Bryan has a lock combination withthree disits. Everynumer in thecombination must be a 0-5what is the probability hiscombination is 000? SopE and SptP have opposing functions in host cell invasion. How does Salmonella use these proteins to orchestrate transient engulfment?A. SopE is injected by SPI1 while SptP is injected later by SPI2B. SopE and SptP are injected simultaneously. SopE temporarily "wins" but is rapidly inactivated by proteolysisC. SopE and SptP are injected simultaneously. SptP temporarily "wins" but is rapidly inactivated by proteolysis.D. SopE and SptP are injected simultaneously. SopE temporarily "wins" but SptP is converted to an active form by proteolysisE. SopE and SptP are injected simultaneously. SopE temporarily "wins" but is rapidly inactivated by glycosylation. Instructions: Read the following case and answer questions stated after the case. (Word limit: 200 words)Khursheed Anwar is 19 years old. He has great interest in IT computer hardware and software, internet, programming languages, and the like. He wants to live a very successful life. His elder cousin, who is studying BBA, has asked him to craft a compelling personal vision as a first step to live a successful life. Khursheed drafted this personal vision: "After 10 years, I want to establish my own advertising company having a large client base, operate a successful travel agency, and run an exclusive boutique for upper-class clients. " He made several goals as per the advice of his "guru cousin. " His cousin had advised him to make goals SMART. He decided to get admission in a leading institute to study ACCA. Another goal that he made is to start learning guitar to impress and entertain relatives and friends in gatherings. He did SWOT analysis as per his cousins advice and in the strength quadrant he had listed computing, programming and physics. Your task:A. What is/are the actual problem/s of Khursheed Anwer? B. What would you suggest Khursheed to rectify these issues based on your knowledge of personal vision, SMART goals, self-esteem and critical thinking? is accounting interesting? Refer to The Mary Celeste: An Unsolved Mystery from History for a complete version of this text.Which detail from the text best supports the idea that the crew of the Mary Celeste may have left the ship alive? I havent been up here for a while but, can I make a song for a video game without being sued? Please dont answer if you dont know. Riverboat Adventures pays $310,000 plus $15,000 in closing costs to purchase real estate. The real estate consists of land appraised at $35,000, a building appraised at $105,000, and land improvements appraised at $210,000. Compute the cost that should be allocated to the land. Opponents of globalization claim that globalization has led to widespread government corruption as indicated through the extensive use of __________. PLEASE HELP!!! ITS EASY!Simplify:(1/2a - 1/3b) - (1/3b - 1/2a) True/False? With the Soviets attacking from the east and with U.S. and British forces attacking Germany from the west, the Allies forced a conditional surrender of Germany on 8 May 1945. Insurance companies face the problem of when people with highest probability of getting the insurance payoffs are the ones who purchase insurance. Multiple Choice moral hazard adverse selection O agency costs O fraud over-insurance Omega construction manufactures homes to customer specifications. it most likely uses:_____. Can someone help me with the questions in the picture? pls help What is the sum? will mark brainiest47+ 85 If one exists, a data modeler uses a _____ as the primary key of the entity being modeled. When the entire network, including all security devices, is virtualized, this is known as:_________ This term emerged in the meetings industry and involves the use of technology to help engage attendees during a meeting.A) GamificationB) Tech GeekC) Smart Phone If the y- and z-coordinates of an ordered triple are both negative, then which statement must be true?A) The point must be located behind the yz-plane.B) The point must be located in front of the yz-plane.C) The point must be located below the xy-plane. D) The point must be located to the right of the xz-plane.